Pt 2-2-q3 Forum

Prepare for the LSAT or discuss it with others in this forum.
Post Reply
User avatar
ltowns1

Silver
Posts: 717
Joined: Mon May 26, 2014 1:13 am

Pt 2-2-q3

Post by ltowns1 » Wed Jul 08, 2015 9:21 pm

I think I understand answer choice (d) now, but I just want some clarity. My initial thinking was that this was a contender because if they were able to do all three, then could it be true that their resources were not limited thus weakening the conclusion. Reviewing this answer has made it a little clearer. This answer is incorrect because it says that you can do all things, but I'm just struggling to see how this has a negative effect on the conclusion?? We want to know about the resources. Being able to do all three things would suggest that your resources are not tied up in drug related crime....I don't know about this answer.

Or maybe it means that your resources are tied up because you're doing all three??? I don't know...
Last edited by ltowns1 on Wed Jul 08, 2015 9:36 pm, edited 1 time in total.

User avatar
RZ5646

Gold
Posts: 2391
Joined: Fri May 30, 2014 1:31 pm

Re: Pt 2-2-q3

Post by RZ5646 » Wed Jul 08, 2015 9:36 pm

D doesn't relate the conclusion and support in the correct way.

Conclusion: it is not the case that they diverted resources from writing speeding tickets to drug crime.

Support for this conclusion: no reduction in writing of speeding tickets

D doesn't really tell us anything relevant. The police could do all 3 things... so what? That doesn't connect the support and conclusion, and the question is asking what assumption is needed, what that missing link is. Perhaps you could say this robs the chief's excuse of its power as an excuse, but it doesn't mean that the content of the excuse is false, which is what the conclusion of this argument is.

E makes the argument deductively valid: diverting resources to drug crime --> not the same number of tickets... contrapositive: same number of tickets --> no diversion of resources to drug crime = chief's excuse is false

User avatar
ltowns1

Silver
Posts: 717
Joined: Mon May 26, 2014 1:13 am

Re: Pt 2-2-q3

Post by ltowns1 » Wed Jul 08, 2015 9:39 pm

RZ5646 wrote:D doesn't relate the conclusion and support in the correct way.

Conclusion: it is not the case that they diverted resources from writing speeding tickets to drug crime.

Support for this conclusion: no reduction in writing of speeding tickets

D doesn't really tell us anything relevant. The police could do all 3 things... so what? That doesn't connect the support and conclusion, and the question is asking what assumption is needed, what that missing link is. Perhaps you could say this robs the chief's excuse of its power as an excuse, but it doesn't mean that the content of the excuse is false, which is what the conclusion of this argument is.

E makes the argument deductively valid: diverting resources to drug crime --> not the same number of tickets... contrapositive: same number of tickets --> no diversion of resources to drug crime = chief's excuse is false

Ok I see that

Post Reply

Return to “LSAT Prep and Discussion Forum”